Is this the most complicated way to construct an equilateral triangle? :)

The name of the pictureThe name of the pictureThe name of the pictureClash Royale CLAN TAG#URR8PPP











up vote
5
down vote

favorite












Just for fun (inspired by sub-problem described and answered here):



Let's pick three points on a circle, say $A,B,C$. Move one point ($A$ for example) until the triangle becomes isosceles ($A'BC$) with all angles acute:



enter image description here



Now we have triangle with sides $AB$ and $AC$ equal. Pick any of the two, say $AC$ and move $B$ until the triangle becomes isosceles again, with all angles acute:



enter image description here



Now we have a triangle with sides $AB$ and $BC$ equal. Pick any of the two, say $BC$ and move $A$ until the triangle becomes isosceles again, with all angles acute:



enter image description here



Repeate the same process infinite times.



Can we prove that the end result is always an equilateral triangle? It looks so but I might be wrong. I have checked several initial configurations and always ended up with something looking like an equilateral triangle.










share|cite|improve this question

























    up vote
    5
    down vote

    favorite












    Just for fun (inspired by sub-problem described and answered here):



    Let's pick three points on a circle, say $A,B,C$. Move one point ($A$ for example) until the triangle becomes isosceles ($A'BC$) with all angles acute:



    enter image description here



    Now we have triangle with sides $AB$ and $AC$ equal. Pick any of the two, say $AC$ and move $B$ until the triangle becomes isosceles again, with all angles acute:



    enter image description here



    Now we have a triangle with sides $AB$ and $BC$ equal. Pick any of the two, say $BC$ and move $A$ until the triangle becomes isosceles again, with all angles acute:



    enter image description here



    Repeate the same process infinite times.



    Can we prove that the end result is always an equilateral triangle? It looks so but I might be wrong. I have checked several initial configurations and always ended up with something looking like an equilateral triangle.










    share|cite|improve this question























      up vote
      5
      down vote

      favorite









      up vote
      5
      down vote

      favorite











      Just for fun (inspired by sub-problem described and answered here):



      Let's pick three points on a circle, say $A,B,C$. Move one point ($A$ for example) until the triangle becomes isosceles ($A'BC$) with all angles acute:



      enter image description here



      Now we have triangle with sides $AB$ and $AC$ equal. Pick any of the two, say $AC$ and move $B$ until the triangle becomes isosceles again, with all angles acute:



      enter image description here



      Now we have a triangle with sides $AB$ and $BC$ equal. Pick any of the two, say $BC$ and move $A$ until the triangle becomes isosceles again, with all angles acute:



      enter image description here



      Repeate the same process infinite times.



      Can we prove that the end result is always an equilateral triangle? It looks so but I might be wrong. I have checked several initial configurations and always ended up with something looking like an equilateral triangle.










      share|cite|improve this question













      Just for fun (inspired by sub-problem described and answered here):



      Let's pick three points on a circle, say $A,B,C$. Move one point ($A$ for example) until the triangle becomes isosceles ($A'BC$) with all angles acute:



      enter image description here



      Now we have triangle with sides $AB$ and $AC$ equal. Pick any of the two, say $AC$ and move $B$ until the triangle becomes isosceles again, with all angles acute:



      enter image description here



      Now we have a triangle with sides $AB$ and $BC$ equal. Pick any of the two, say $BC$ and move $A$ until the triangle becomes isosceles again, with all angles acute:



      enter image description here



      Repeate the same process infinite times.



      Can we prove that the end result is always an equilateral triangle? It looks so but I might be wrong. I have checked several initial configurations and always ended up with something looking like an equilateral triangle.







      euclidean-geometry triangle






      share|cite|improve this question













      share|cite|improve this question











      share|cite|improve this question




      share|cite|improve this question










      asked 3 hours ago









      Oldboy

      3,1931321




      3,1931321




















          3 Answers
          3






          active

          oldest

          votes

















          up vote
          3
          down vote













          Think about what happens to the maximum difference between angles over time.



          For simplicity, let's start with an isoceles triangle with angles $x,y,y$. This triangle has "maximum angle difference" $vert y-xvert$. Then when we move one of the $y$-angled points, our new triangle will have angles



          $$y, x+yover 2, x+yover 2$$



          since the angle of the point being moved doesn't change.The maximum difference of angles in this new triangle is



          $$leftvert yover 2-xover 2rightvert=1over 2vert y-xvert.$$



          So each time we perform this transformation, the maximum angle difference goes down by a factor of two. Whatever the initial value $vert y-xvert$ was, this means that the maximum angle difference goes to zero,$^*$ which in turn means that in the limit the angles are equal.




          $^*$This is because it's a geometric sequence with ratio in $(-1,1)$ (namely, $1over 2$): if $rin(-1,1)$ then for any $a$ we have



          $$lim_nrightarrowinftyar^n=0.$$



          Note that it would not have been enough to simply know that the maximum angle difference decreases, since not every decreasing sequence goes to zero!






          share|cite|improve this answer





























            up vote
            1
            down vote













            Assume WLOG that the initial triangle is isoceles. Let $alpha$ be the apical angle, and let $beta$ be a remaining angle. Then the transformation in question sends



            $$beginbmatrixalpha \ beta endbmatrixmapsto
            beginbmatrix0 & 1 \ tfrac12 & tfrac12
            endbmatrix
            beginbmatrixalpha \ beta endbmatrixtext.$$
            Let $mathsfX$ be the $2times 2$ transformation matrix on the rhs. $mathsfX$ has characteristic polynomial
            $x^2-tfrac12x-tfrac12=0.$ By the Cayley–Hamilton theorem,
            $$mathsfX^2=tfrac12mathsfX+tfrac12text.$$
            Therefore we have a Sylvester formula
            $$f(mathsfX)=f(1)left(frac1+2mathsfX3right)+f(-tfrac12)left(frac2-2mathsfX3right)$$
            for any polynomial $f$ (where the matrices in brackets are the Frobenius covariants). Thus,
            $$mathsfX^n=frac1+2mathsfX3+(-tfrac12)^nleft(frac2-2mathsfX3right)text.$$
            The second term converges to zero, so
            $$beginsplit
            lim_ntoinftymathsfX^n&=frac1+2mathsfX3\
            &=frac13beginbmatrix1 & 2 \ 1 & 2endbmatrix\
            &=frac13beginbmatrix 1\ 1endbmatrixbeginbmatrix1&2endbmatrixtext,
            endsplit$$
            $$lim_ntoinfty
            beginbmatrix0 & 1 \ tfrac12 & tfrac12
            endbmatrix^n
            beginbmatrixalpha \ beta endbmatrix=beginbmatrixtfracalpha+2beta3\ tfracalpha+2beta3endbmatrixtext.$$
            i.e., the apical and side angles approach equality as the operation is repeated.






            share|cite|improve this answer



























              up vote
              1
              down vote













              By the Inscribed Angle Theorem, moving a vertex around the circle preserves the angle at that vertex. Now, suppose that, at stage $i$, the apex angle is $theta_i$, so that the base angles are $frac12(pi - theta_i)$. But this apex angle was the base angle of the previous step, giving this recurrence $theta_i = frac12(pi-theta_i-1)$. Thus,
              $$beginaligntheta_n &= -frac12theta_n-1 + frac12pi \[6pt]
              &=frac12left(-frac12(pi-theta_n-2)+piright) = frac14theta_n-2+frac12pi-frac14pi \[6pt]
              &= cdots \[6pt]
              &= left(-frac12right)^ntheta_0 ;-; sum_i=1^nleft(-frac12right)^npi \[6pt]
              lim_ntoinftytheta_n &= 0cdottheta_0 ;-; frac(-1/2)1-(-1/2)pi \
              &=fracpi3
              endalign$$



              Thus, in the limit, the triangle becomes equilateral. $square$






              share|cite|improve this answer




















                Your Answer




                StackExchange.ifUsing("editor", function ()
                return StackExchange.using("mathjaxEditing", function ()
                StackExchange.MarkdownEditor.creationCallbacks.add(function (editor, postfix)
                StackExchange.mathjaxEditing.prepareWmdForMathJax(editor, postfix, [["$", "$"], ["\\(","\\)"]]);
                );
                );
                , "mathjax-editing");

                StackExchange.ready(function()
                var channelOptions =
                tags: "".split(" "),
                id: "69"
                ;
                initTagRenderer("".split(" "), "".split(" "), channelOptions);

                StackExchange.using("externalEditor", function()
                // Have to fire editor after snippets, if snippets enabled
                if (StackExchange.settings.snippets.snippetsEnabled)
                StackExchange.using("snippets", function()
                createEditor();
                );

                else
                createEditor();

                );

                function createEditor()
                StackExchange.prepareEditor(
                heartbeatType: 'answer',
                convertImagesToLinks: true,
                noModals: false,
                showLowRepImageUploadWarning: true,
                reputationToPostImages: 10,
                bindNavPrevention: true,
                postfix: "",
                noCode: true, onDemand: true,
                discardSelector: ".discard-answer"
                ,immediatelyShowMarkdownHelp:true
                );



                );













                 

                draft saved


                draft discarded


















                StackExchange.ready(
                function ()
                StackExchange.openid.initPostLogin('.new-post-login', 'https%3a%2f%2fmath.stackexchange.com%2fquestions%2f2920725%2fis-this-the-most-complicated-way-to-construct-an-equilateral-triangle%23new-answer', 'question_page');

                );

                Post as a guest






























                3 Answers
                3






                active

                oldest

                votes








                3 Answers
                3






                active

                oldest

                votes









                active

                oldest

                votes






                active

                oldest

                votes








                up vote
                3
                down vote













                Think about what happens to the maximum difference between angles over time.



                For simplicity, let's start with an isoceles triangle with angles $x,y,y$. This triangle has "maximum angle difference" $vert y-xvert$. Then when we move one of the $y$-angled points, our new triangle will have angles



                $$y, x+yover 2, x+yover 2$$



                since the angle of the point being moved doesn't change.The maximum difference of angles in this new triangle is



                $$leftvert yover 2-xover 2rightvert=1over 2vert y-xvert.$$



                So each time we perform this transformation, the maximum angle difference goes down by a factor of two. Whatever the initial value $vert y-xvert$ was, this means that the maximum angle difference goes to zero,$^*$ which in turn means that in the limit the angles are equal.




                $^*$This is because it's a geometric sequence with ratio in $(-1,1)$ (namely, $1over 2$): if $rin(-1,1)$ then for any $a$ we have



                $$lim_nrightarrowinftyar^n=0.$$



                Note that it would not have been enough to simply know that the maximum angle difference decreases, since not every decreasing sequence goes to zero!






                share|cite|improve this answer


























                  up vote
                  3
                  down vote













                  Think about what happens to the maximum difference between angles over time.



                  For simplicity, let's start with an isoceles triangle with angles $x,y,y$. This triangle has "maximum angle difference" $vert y-xvert$. Then when we move one of the $y$-angled points, our new triangle will have angles



                  $$y, x+yover 2, x+yover 2$$



                  since the angle of the point being moved doesn't change.The maximum difference of angles in this new triangle is



                  $$leftvert yover 2-xover 2rightvert=1over 2vert y-xvert.$$



                  So each time we perform this transformation, the maximum angle difference goes down by a factor of two. Whatever the initial value $vert y-xvert$ was, this means that the maximum angle difference goes to zero,$^*$ which in turn means that in the limit the angles are equal.




                  $^*$This is because it's a geometric sequence with ratio in $(-1,1)$ (namely, $1over 2$): if $rin(-1,1)$ then for any $a$ we have



                  $$lim_nrightarrowinftyar^n=0.$$



                  Note that it would not have been enough to simply know that the maximum angle difference decreases, since not every decreasing sequence goes to zero!






                  share|cite|improve this answer
























                    up vote
                    3
                    down vote










                    up vote
                    3
                    down vote









                    Think about what happens to the maximum difference between angles over time.



                    For simplicity, let's start with an isoceles triangle with angles $x,y,y$. This triangle has "maximum angle difference" $vert y-xvert$. Then when we move one of the $y$-angled points, our new triangle will have angles



                    $$y, x+yover 2, x+yover 2$$



                    since the angle of the point being moved doesn't change.The maximum difference of angles in this new triangle is



                    $$leftvert yover 2-xover 2rightvert=1over 2vert y-xvert.$$



                    So each time we perform this transformation, the maximum angle difference goes down by a factor of two. Whatever the initial value $vert y-xvert$ was, this means that the maximum angle difference goes to zero,$^*$ which in turn means that in the limit the angles are equal.




                    $^*$This is because it's a geometric sequence with ratio in $(-1,1)$ (namely, $1over 2$): if $rin(-1,1)$ then for any $a$ we have



                    $$lim_nrightarrowinftyar^n=0.$$



                    Note that it would not have been enough to simply know that the maximum angle difference decreases, since not every decreasing sequence goes to zero!






                    share|cite|improve this answer














                    Think about what happens to the maximum difference between angles over time.



                    For simplicity, let's start with an isoceles triangle with angles $x,y,y$. This triangle has "maximum angle difference" $vert y-xvert$. Then when we move one of the $y$-angled points, our new triangle will have angles



                    $$y, x+yover 2, x+yover 2$$



                    since the angle of the point being moved doesn't change.The maximum difference of angles in this new triangle is



                    $$leftvert yover 2-xover 2rightvert=1over 2vert y-xvert.$$



                    So each time we perform this transformation, the maximum angle difference goes down by a factor of two. Whatever the initial value $vert y-xvert$ was, this means that the maximum angle difference goes to zero,$^*$ which in turn means that in the limit the angles are equal.




                    $^*$This is because it's a geometric sequence with ratio in $(-1,1)$ (namely, $1over 2$): if $rin(-1,1)$ then for any $a$ we have



                    $$lim_nrightarrowinftyar^n=0.$$



                    Note that it would not have been enough to simply know that the maximum angle difference decreases, since not every decreasing sequence goes to zero!







                    share|cite|improve this answer














                    share|cite|improve this answer



                    share|cite|improve this answer








                    edited 1 hour ago

























                    answered 2 hours ago









                    Noah Schweber

                    112k9142266




                    112k9142266




















                        up vote
                        1
                        down vote













                        Assume WLOG that the initial triangle is isoceles. Let $alpha$ be the apical angle, and let $beta$ be a remaining angle. Then the transformation in question sends



                        $$beginbmatrixalpha \ beta endbmatrixmapsto
                        beginbmatrix0 & 1 \ tfrac12 & tfrac12
                        endbmatrix
                        beginbmatrixalpha \ beta endbmatrixtext.$$
                        Let $mathsfX$ be the $2times 2$ transformation matrix on the rhs. $mathsfX$ has characteristic polynomial
                        $x^2-tfrac12x-tfrac12=0.$ By the Cayley–Hamilton theorem,
                        $$mathsfX^2=tfrac12mathsfX+tfrac12text.$$
                        Therefore we have a Sylvester formula
                        $$f(mathsfX)=f(1)left(frac1+2mathsfX3right)+f(-tfrac12)left(frac2-2mathsfX3right)$$
                        for any polynomial $f$ (where the matrices in brackets are the Frobenius covariants). Thus,
                        $$mathsfX^n=frac1+2mathsfX3+(-tfrac12)^nleft(frac2-2mathsfX3right)text.$$
                        The second term converges to zero, so
                        $$beginsplit
                        lim_ntoinftymathsfX^n&=frac1+2mathsfX3\
                        &=frac13beginbmatrix1 & 2 \ 1 & 2endbmatrix\
                        &=frac13beginbmatrix 1\ 1endbmatrixbeginbmatrix1&2endbmatrixtext,
                        endsplit$$
                        $$lim_ntoinfty
                        beginbmatrix0 & 1 \ tfrac12 & tfrac12
                        endbmatrix^n
                        beginbmatrixalpha \ beta endbmatrix=beginbmatrixtfracalpha+2beta3\ tfracalpha+2beta3endbmatrixtext.$$
                        i.e., the apical and side angles approach equality as the operation is repeated.






                        share|cite|improve this answer
























                          up vote
                          1
                          down vote













                          Assume WLOG that the initial triangle is isoceles. Let $alpha$ be the apical angle, and let $beta$ be a remaining angle. Then the transformation in question sends



                          $$beginbmatrixalpha \ beta endbmatrixmapsto
                          beginbmatrix0 & 1 \ tfrac12 & tfrac12
                          endbmatrix
                          beginbmatrixalpha \ beta endbmatrixtext.$$
                          Let $mathsfX$ be the $2times 2$ transformation matrix on the rhs. $mathsfX$ has characteristic polynomial
                          $x^2-tfrac12x-tfrac12=0.$ By the Cayley–Hamilton theorem,
                          $$mathsfX^2=tfrac12mathsfX+tfrac12text.$$
                          Therefore we have a Sylvester formula
                          $$f(mathsfX)=f(1)left(frac1+2mathsfX3right)+f(-tfrac12)left(frac2-2mathsfX3right)$$
                          for any polynomial $f$ (where the matrices in brackets are the Frobenius covariants). Thus,
                          $$mathsfX^n=frac1+2mathsfX3+(-tfrac12)^nleft(frac2-2mathsfX3right)text.$$
                          The second term converges to zero, so
                          $$beginsplit
                          lim_ntoinftymathsfX^n&=frac1+2mathsfX3\
                          &=frac13beginbmatrix1 & 2 \ 1 & 2endbmatrix\
                          &=frac13beginbmatrix 1\ 1endbmatrixbeginbmatrix1&2endbmatrixtext,
                          endsplit$$
                          $$lim_ntoinfty
                          beginbmatrix0 & 1 \ tfrac12 & tfrac12
                          endbmatrix^n
                          beginbmatrixalpha \ beta endbmatrix=beginbmatrixtfracalpha+2beta3\ tfracalpha+2beta3endbmatrixtext.$$
                          i.e., the apical and side angles approach equality as the operation is repeated.






                          share|cite|improve this answer






















                            up vote
                            1
                            down vote










                            up vote
                            1
                            down vote









                            Assume WLOG that the initial triangle is isoceles. Let $alpha$ be the apical angle, and let $beta$ be a remaining angle. Then the transformation in question sends



                            $$beginbmatrixalpha \ beta endbmatrixmapsto
                            beginbmatrix0 & 1 \ tfrac12 & tfrac12
                            endbmatrix
                            beginbmatrixalpha \ beta endbmatrixtext.$$
                            Let $mathsfX$ be the $2times 2$ transformation matrix on the rhs. $mathsfX$ has characteristic polynomial
                            $x^2-tfrac12x-tfrac12=0.$ By the Cayley–Hamilton theorem,
                            $$mathsfX^2=tfrac12mathsfX+tfrac12text.$$
                            Therefore we have a Sylvester formula
                            $$f(mathsfX)=f(1)left(frac1+2mathsfX3right)+f(-tfrac12)left(frac2-2mathsfX3right)$$
                            for any polynomial $f$ (where the matrices in brackets are the Frobenius covariants). Thus,
                            $$mathsfX^n=frac1+2mathsfX3+(-tfrac12)^nleft(frac2-2mathsfX3right)text.$$
                            The second term converges to zero, so
                            $$beginsplit
                            lim_ntoinftymathsfX^n&=frac1+2mathsfX3\
                            &=frac13beginbmatrix1 & 2 \ 1 & 2endbmatrix\
                            &=frac13beginbmatrix 1\ 1endbmatrixbeginbmatrix1&2endbmatrixtext,
                            endsplit$$
                            $$lim_ntoinfty
                            beginbmatrix0 & 1 \ tfrac12 & tfrac12
                            endbmatrix^n
                            beginbmatrixalpha \ beta endbmatrix=beginbmatrixtfracalpha+2beta3\ tfracalpha+2beta3endbmatrixtext.$$
                            i.e., the apical and side angles approach equality as the operation is repeated.






                            share|cite|improve this answer












                            Assume WLOG that the initial triangle is isoceles. Let $alpha$ be the apical angle, and let $beta$ be a remaining angle. Then the transformation in question sends



                            $$beginbmatrixalpha \ beta endbmatrixmapsto
                            beginbmatrix0 & 1 \ tfrac12 & tfrac12
                            endbmatrix
                            beginbmatrixalpha \ beta endbmatrixtext.$$
                            Let $mathsfX$ be the $2times 2$ transformation matrix on the rhs. $mathsfX$ has characteristic polynomial
                            $x^2-tfrac12x-tfrac12=0.$ By the Cayley–Hamilton theorem,
                            $$mathsfX^2=tfrac12mathsfX+tfrac12text.$$
                            Therefore we have a Sylvester formula
                            $$f(mathsfX)=f(1)left(frac1+2mathsfX3right)+f(-tfrac12)left(frac2-2mathsfX3right)$$
                            for any polynomial $f$ (where the matrices in brackets are the Frobenius covariants). Thus,
                            $$mathsfX^n=frac1+2mathsfX3+(-tfrac12)^nleft(frac2-2mathsfX3right)text.$$
                            The second term converges to zero, so
                            $$beginsplit
                            lim_ntoinftymathsfX^n&=frac1+2mathsfX3\
                            &=frac13beginbmatrix1 & 2 \ 1 & 2endbmatrix\
                            &=frac13beginbmatrix 1\ 1endbmatrixbeginbmatrix1&2endbmatrixtext,
                            endsplit$$
                            $$lim_ntoinfty
                            beginbmatrix0 & 1 \ tfrac12 & tfrac12
                            endbmatrix^n
                            beginbmatrixalpha \ beta endbmatrix=beginbmatrixtfracalpha+2beta3\ tfracalpha+2beta3endbmatrixtext.$$
                            i.e., the apical and side angles approach equality as the operation is repeated.







                            share|cite|improve this answer












                            share|cite|improve this answer



                            share|cite|improve this answer










                            answered 2 hours ago









                            K B Dave

                            2,744216




                            2,744216




















                                up vote
                                1
                                down vote













                                By the Inscribed Angle Theorem, moving a vertex around the circle preserves the angle at that vertex. Now, suppose that, at stage $i$, the apex angle is $theta_i$, so that the base angles are $frac12(pi - theta_i)$. But this apex angle was the base angle of the previous step, giving this recurrence $theta_i = frac12(pi-theta_i-1)$. Thus,
                                $$beginaligntheta_n &= -frac12theta_n-1 + frac12pi \[6pt]
                                &=frac12left(-frac12(pi-theta_n-2)+piright) = frac14theta_n-2+frac12pi-frac14pi \[6pt]
                                &= cdots \[6pt]
                                &= left(-frac12right)^ntheta_0 ;-; sum_i=1^nleft(-frac12right)^npi \[6pt]
                                lim_ntoinftytheta_n &= 0cdottheta_0 ;-; frac(-1/2)1-(-1/2)pi \
                                &=fracpi3
                                endalign$$



                                Thus, in the limit, the triangle becomes equilateral. $square$






                                share|cite|improve this answer
























                                  up vote
                                  1
                                  down vote













                                  By the Inscribed Angle Theorem, moving a vertex around the circle preserves the angle at that vertex. Now, suppose that, at stage $i$, the apex angle is $theta_i$, so that the base angles are $frac12(pi - theta_i)$. But this apex angle was the base angle of the previous step, giving this recurrence $theta_i = frac12(pi-theta_i-1)$. Thus,
                                  $$beginaligntheta_n &= -frac12theta_n-1 + frac12pi \[6pt]
                                  &=frac12left(-frac12(pi-theta_n-2)+piright) = frac14theta_n-2+frac12pi-frac14pi \[6pt]
                                  &= cdots \[6pt]
                                  &= left(-frac12right)^ntheta_0 ;-; sum_i=1^nleft(-frac12right)^npi \[6pt]
                                  lim_ntoinftytheta_n &= 0cdottheta_0 ;-; frac(-1/2)1-(-1/2)pi \
                                  &=fracpi3
                                  endalign$$



                                  Thus, in the limit, the triangle becomes equilateral. $square$






                                  share|cite|improve this answer






















                                    up vote
                                    1
                                    down vote










                                    up vote
                                    1
                                    down vote









                                    By the Inscribed Angle Theorem, moving a vertex around the circle preserves the angle at that vertex. Now, suppose that, at stage $i$, the apex angle is $theta_i$, so that the base angles are $frac12(pi - theta_i)$. But this apex angle was the base angle of the previous step, giving this recurrence $theta_i = frac12(pi-theta_i-1)$. Thus,
                                    $$beginaligntheta_n &= -frac12theta_n-1 + frac12pi \[6pt]
                                    &=frac12left(-frac12(pi-theta_n-2)+piright) = frac14theta_n-2+frac12pi-frac14pi \[6pt]
                                    &= cdots \[6pt]
                                    &= left(-frac12right)^ntheta_0 ;-; sum_i=1^nleft(-frac12right)^npi \[6pt]
                                    lim_ntoinftytheta_n &= 0cdottheta_0 ;-; frac(-1/2)1-(-1/2)pi \
                                    &=fracpi3
                                    endalign$$



                                    Thus, in the limit, the triangle becomes equilateral. $square$






                                    share|cite|improve this answer












                                    By the Inscribed Angle Theorem, moving a vertex around the circle preserves the angle at that vertex. Now, suppose that, at stage $i$, the apex angle is $theta_i$, so that the base angles are $frac12(pi - theta_i)$. But this apex angle was the base angle of the previous step, giving this recurrence $theta_i = frac12(pi-theta_i-1)$. Thus,
                                    $$beginaligntheta_n &= -frac12theta_n-1 + frac12pi \[6pt]
                                    &=frac12left(-frac12(pi-theta_n-2)+piright) = frac14theta_n-2+frac12pi-frac14pi \[6pt]
                                    &= cdots \[6pt]
                                    &= left(-frac12right)^ntheta_0 ;-; sum_i=1^nleft(-frac12right)^npi \[6pt]
                                    lim_ntoinftytheta_n &= 0cdottheta_0 ;-; frac(-1/2)1-(-1/2)pi \
                                    &=fracpi3
                                    endalign$$



                                    Thus, in the limit, the triangle becomes equilateral. $square$







                                    share|cite|improve this answer












                                    share|cite|improve this answer



                                    share|cite|improve this answer










                                    answered 2 hours ago









                                    Blue

                                    44.2k868141




                                    44.2k868141



























                                         

                                        draft saved


                                        draft discarded















































                                         


                                        draft saved


                                        draft discarded














                                        StackExchange.ready(
                                        function ()
                                        StackExchange.openid.initPostLogin('.new-post-login', 'https%3a%2f%2fmath.stackexchange.com%2fquestions%2f2920725%2fis-this-the-most-complicated-way-to-construct-an-equilateral-triangle%23new-answer', 'question_page');

                                        );

                                        Post as a guest













































































                                        Comments

                                        Popular posts from this blog

                                        Long meetings (6-7 hours a day): Being “babysat” by supervisor

                                        Is the Concept of Multiple Fantasy Races Scientifically Flawed? [closed]

                                        Confectionery